Question and Answers Forum

All Questions      Topic List

Others Questions

Previous in All Question      Next in All Question      

Previous in Others      Next in Others      

Question Number 25569 by rita1608 last updated on 11/Dec/17

if y =(sin^(−1) x)^2 ,check whether  (1−x^2 )y_(n+2) −(2n+1)xy_(n+1) +n^2 y_n =0  or not.

$${if}\:{y}\:=\left({sin}^{−\mathrm{1}} {x}\right)^{\mathrm{2}} ,{check}\:{whether} \\ $$$$\left(\mathrm{1}−{x}^{\mathrm{2}} \right){y}_{{n}+\mathrm{2}} −\left(\mathrm{2}{n}+\mathrm{1}\right){xy}_{{n}+\mathrm{1}} +{n}^{\mathrm{2}} {y}_{{n}} =\mathrm{0} \\ $$$${or}\:{not}. \\ $$

Commented by prakash jain last updated on 12/Dec/17

yes. The ans is no.

$${yes}.\:\mathrm{The}\:\mathrm{ans}\:\mathrm{is}\:\mathrm{no}. \\ $$

Commented by rita1608 last updated on 11/Dec/17

in this question i am getting ans   (1−x^2 )y_(n+2) (2n−1)xy_(n+1) −n^2 y_n =0  i just want to know why i am getting   − ve sign before n^2 y_(n )

$${in}\:{this}\:{question}\:{i}\:{am}\:{getting}\:{ans}\: \\ $$$$\left(\mathrm{1}−{x}^{\mathrm{2}} \right){y}_{{n}+\mathrm{2}} \left(\mathrm{2}{n}−\mathrm{1}\right){xy}_{{n}+\mathrm{1}} −{n}^{\mathrm{2}} {y}_{{n}} =\mathrm{0} \\ $$$${i}\:{just}\:{want}\:{to}\:{know}\:{why}\:{i}\:{am}\:{getting}\: \\ $$$$−\:{ve}\:{sign}\:{before}\:{n}^{\mathrm{2}} {y}_{{n}\:} \\ $$

Commented by prakash jain last updated on 11/Dec/17

Can u please share your workings?

$$\mathrm{Can}\:\mathrm{u}\:\mathrm{please}\:\mathrm{share}\:\mathrm{your}\:\mathrm{workings}? \\ $$

Commented by rita1608 last updated on 12/Dec/17

ok

$${ok}\: \\ $$

Commented by prakash jain last updated on 12/Dec/17

y=(sin^(−1) x)^2   y′=((2sin^(−1) x)/(√(1−x^2 )))  y′′=(2/((1−x^2 )))+((2xsin^(−1) x)/((1−x^2 )^(3/2) ))  y_3 =((6x)/((1−x^2 )^2 ))+((6x^2 sin^(−1) x)/((1−x^2 )^(5/2) ))+((2sin^(−1) x)/((1−x^2 )^(3/2) ))  (1−x^2 )y_3 −(2n+1)xy_2 −n^2 y_1   =((6x)/((1−x^2 )))+((6x^2 sin^(−1) x)/((1−x^2 )^(3/2) ))+((2sin^(−1) x)/((1−x^2 )^(1/2) ))      −((6x)/((1−x^2 )))−((6x^2 sin^(−1) x)/((1−x^2 )^(3/2) ))      −((2sin^(−1) x)/(√(1−x^2 )))=0  I am also getting −n^2  for n=1

$${y}=\left(\mathrm{sin}^{−\mathrm{1}} {x}\right)^{\mathrm{2}} \\ $$$${y}'=\frac{\mathrm{2sin}^{−\mathrm{1}} {x}}{\sqrt{\mathrm{1}−{x}^{\mathrm{2}} }} \\ $$$${y}''=\frac{\mathrm{2}}{\left(\mathrm{1}−{x}^{\mathrm{2}} \right)}+\frac{\mathrm{2}{x}\mathrm{sin}^{−\mathrm{1}} {x}}{\left(\mathrm{1}−{x}^{\mathrm{2}} \right)^{\mathrm{3}/\mathrm{2}} } \\ $$$${y}_{\mathrm{3}} =\frac{\mathrm{6}{x}}{\left(\mathrm{1}−{x}^{\mathrm{2}} \right)^{\mathrm{2}} }+\frac{\mathrm{6}{x}^{\mathrm{2}} \mathrm{sin}^{−\mathrm{1}} {x}}{\left(\mathrm{1}−{x}^{\mathrm{2}} \right)^{\mathrm{5}/\mathrm{2}} }+\frac{\mathrm{2sin}^{−\mathrm{1}} {x}}{\left(\mathrm{1}−{x}^{\mathrm{2}} \right)^{\mathrm{3}/\mathrm{2}} } \\ $$$$\left(\mathrm{1}−{x}^{\mathrm{2}} \right){y}_{\mathrm{3}} −\left(\mathrm{2}{n}+\mathrm{1}\right){xy}_{\mathrm{2}} −{n}^{\mathrm{2}} {y}_{\mathrm{1}} \\ $$$$=\frac{\mathrm{6}{x}}{\left(\mathrm{1}−{x}^{\mathrm{2}} \right)}+\frac{\mathrm{6}{x}^{\mathrm{2}} \mathrm{sin}^{−\mathrm{1}} {x}}{\left(\mathrm{1}−{x}^{\mathrm{2}} \right)^{\mathrm{3}/\mathrm{2}} }+\frac{\mathrm{2sin}^{−\mathrm{1}} {x}}{\left(\mathrm{1}−{x}^{\mathrm{2}} \right)^{\mathrm{1}/\mathrm{2}} } \\ $$$$\:\:\:\:−\frac{\mathrm{6}{x}}{\left(\mathrm{1}−{x}^{\mathrm{2}} \right)}−\frac{\mathrm{6}{x}^{\mathrm{2}} \mathrm{sin}^{−\mathrm{1}} {x}}{\left(\mathrm{1}−{x}^{\mathrm{2}} \right)^{\mathrm{3}/\mathrm{2}} } \\ $$$$\:\:\:\:−\frac{\mathrm{2sin}^{−\mathrm{1}} {x}}{\sqrt{\mathrm{1}−{x}^{\mathrm{2}} }}=\mathrm{0} \\ $$$$\mathrm{I}\:\mathrm{am}\:\mathrm{also}\:\mathrm{getting}\:−{n}^{\mathrm{2}} \:\mathrm{for}\:{n}=\mathrm{1} \\ $$

Commented by rita1608 last updated on 12/Dec/17

Commented by prakash jain last updated on 12/Dec/17

It looks correct. Also i checked  calculation with y_3 , y_2 ,y_1  it is  −n^2 .

$$\mathrm{It}\:\mathrm{looks}\:\mathrm{correct}.\:\mathrm{Also}\:\mathrm{i}\:\mathrm{checked} \\ $$$$\mathrm{calculation}\:\mathrm{with}\:{y}_{\mathrm{3}} ,\:{y}_{\mathrm{2}} ,{y}_{\mathrm{1}} \:\mathrm{it}\:\mathrm{is} \\ $$$$−{n}^{\mathrm{2}} . \\ $$

Commented by rita1608 last updated on 12/Dec/17

ok but in question they are saying to   check.then ans is no ?

$${ok}\:{but}\:{in}\:{question}\:{they}\:{are}\:{saying}\:{to}\: \\ $$$${check}.{then}\:{ans}\:{is}\:{no}\:? \\ $$

Terms of Service

Privacy Policy

Contact: info@tinkutara.com